Mathcenter Forum  

Go Back   Mathcenter Forum > คณิตศาสตร์มัธยมศึกษา > ปัญหาคณิตศาสตร์ ม. ต้น
สมัครสมาชิก คู่มือการใช้ รายชื่อสมาชิก ปฏิทิน ข้อความวันนี้

ตั้งหัวข้อใหม่ Reply
 
เครื่องมือของหัวข้อ ค้นหาในหัวข้อนี้
  #1  
Old 23 กรกฎาคม 2008, 22:15
Yo WMU Yo WMU ไม่อยู่ในระบบ
ลมปราณคุ้มครองร่าง
 
วันที่สมัครสมาชิก: 15 กรกฎาคม 2008
ข้อความ: 265
Yo WMU is on a distinguished road
Default ช่วยผมคิด 2 ข้อนี้หน่อยครับ

1) ถ้า a, b, c เป็นรากทั้งสามของสมการ x^3 - x^2 + x - 2 = 0
แล้ว a^3 + b^3 + c^3 มีค่าเท่ากับเท่าไร

2) มีจำนวนเต็ม m, n กี่คู่ที่สอดคล้องกับสมการ m^3 + 6m^2 + 5m = 27n^3 + 9n^2 + 9n + 1
ตอบพร้อมอ้างอิงข้อความนี้
  #2  
Old 23 กรกฎาคม 2008, 22:52
jabza's Avatar
jabza jabza ไม่อยู่ในระบบ
บัณฑิตฟ้า
 
วันที่สมัครสมาชิก: 02 สิงหาคม 2005
ข้อความ: 544
jabza is on a distinguished road
Default

ข้อ1ตอบ=4 hint. a+b+c=1,ab+bc+ca=1,abc=2, สมการ a^3+b^3+c^3=(a+b+c)(a^2+b^+c^-ab-bc-ca)+3abc แทนค่า จะได้=(1)*(-2)*(+6)= 4
__________________
จะขอทำฝัน....ให้ใกล้เคียงความจริงที่สุด

เด็กน้อย ค่อยๆ เรียนรู้ สินะ
ตอบพร้อมอ้างอิงข้อความนี้
  #3  
Old 23 กรกฎาคม 2008, 23:53
Yo WMU Yo WMU ไม่อยู่ในระบบ
ลมปราณคุ้มครองร่าง
 
วันที่สมัครสมาชิก: 15 กรกฎาคม 2008
ข้อความ: 265
Yo WMU is on a distinguished road
Default

โอ้ thank you ครับคุณ Jabza ผมเข้าใจละครับ
ตอบพร้อมอ้างอิงข้อความนี้
  #4  
Old 24 กรกฎาคม 2008, 07:09
jabza's Avatar
jabza jabza ไม่อยู่ในระบบ
บัณฑิตฟ้า
 
วันที่สมัครสมาชิก: 02 สิงหาคม 2005
ข้อความ: 544
jabza is on a distinguished road
Default

ข้อ2นี้น่าสนใจมาก(สงสัยโจทย์สมการขวามือ แยกตัวประกอบไม่ได้) ผมยังคิดไม่ออก. ต้องให้คนเก่งคณิตมาช่วยเฉลย.
__________________
จะขอทำฝัน....ให้ใกล้เคียงความจริงที่สุด

เด็กน้อย ค่อยๆ เรียนรู้ สินะ
ตอบพร้อมอ้างอิงข้อความนี้
  #5  
Old 26 กรกฎาคม 2008, 19:11
MirRor's Avatar
MirRor MirRor ไม่อยู่ในระบบ
บัณฑิตฟ้า
 
วันที่สมัครสมาชิก: 03 มีนาคม 2008
ข้อความ: 394
MirRor is on a distinguished road
Default

ข้อสองนี้การหารสังเคราะห์ใช้ได้ป่ะ

- -
__________________

26 กรกฎาคม 2008 19:22 : ข้อความนี้ถูกแก้ไขแล้ว 1 ครั้ง, ครั้งล่าสุดโดยคุณ MirRor
ตอบพร้อมอ้างอิงข้อความนี้
  #6  
Old 26 กรกฎาคม 2008, 19:21
หยินหยาง's Avatar
หยินหยาง หยินหยาง ไม่อยู่ในระบบ
กระบี่จักรวาล
 
วันที่สมัครสมาชิก: 06 มกราคม 2007
ข้อความ: 2,921
หยินหยาง is on a distinguished road
Default

อ้างอิง:
ข้อความเดิมเขียนโดยคุณ MirRor View Post
ข้อสองนี้ลองใช้ทฤษฎีบทเศษเหลือกับการหารสังเคราะห์เข้ามาช่วยด้วยนะครับ พอดีพึ่งไปเรียนมา

- -
งั้นลองช่วยแสดงวิธีทำให้ดูหน่อยได้มั้ยครับ เผื่อผมจะได้หู ตา สว่างขึ้น
ตอบพร้อมอ้างอิงข้อความนี้
  #7  
Old 26 กรกฎาคม 2008, 19:35
MirRor's Avatar
MirRor MirRor ไม่อยู่ในระบบ
บัณฑิตฟ้า
 
วันที่สมัครสมาชิก: 03 มีนาคม 2008
ข้อความ: 394
MirRor is on a distinguished road
Default

อ่าโทษที คงไม่ได้และ - -

เราติดที่ m[(m+3)(m+2)]-1 = 9n($3n^2$+n+1)

ขอผู้รู้ช่วยชี้แนะที
__________________
ตอบพร้อมอ้างอิงข้อความนี้
  #8  
Old 26 กรกฎาคม 2008, 20:52
หยินหยาง's Avatar
หยินหยาง หยินหยาง ไม่อยู่ในระบบ
กระบี่จักรวาล
 
วันที่สมัครสมาชิก: 06 มกราคม 2007
ข้อความ: 2,921
หยินหยาง is on a distinguished road
Default

อ้างอิง:
ข้อความเดิมเขียนโดยคุณ MirRor View Post
อ่าโทษที คงไม่ได้และ - -

เราติดที่ m[(m+3)(m+2)]-1 = 9n($3n^2$+n+1)

ขอผู้รู้ช่วยชี้แนะที
พจน์ซ้ายมือถ้าจะแยก น่าจะเป็นอย่างนี้จะดูง่ายกว่าครับ $m(m+1)(m+5)$ ส่วนพจน์ทางขวามือจะแยกตัวประกอบคงไม่ง่ายนัก แต่ถ้าแยกได้สำคัญว่าจะไปใช้อะไรอย่างไรต่อไป
ประเด็นของข้อนี้อยู่ทีว่าจะมีจำนวนเต็มกี่ชุดที่ทำให้พจน์ทางซ้ายเท่ากับทางขวา ข้อนี้คงต้องพิจารณาเป็นช่วงและดูลักษณะของกราฟประกอบด้วย
ตอบพร้อมอ้างอิงข้อความนี้
  #9  
Old 26 กรกฎาคม 2008, 21:00
MirRor's Avatar
MirRor MirRor ไม่อยู่ในระบบ
บัณฑิตฟ้า
 
วันที่สมัครสมาชิก: 03 มีนาคม 2008
ข้อความ: 394
MirRor is on a distinguished road
Default

อ่าใช่ เราแยกตัวประกอบผิดเอง แฮะๆ
__________________
ตอบพร้อมอ้างอิงข้อความนี้
  #10  
Old 26 กรกฎาคม 2008, 22:30
jabza's Avatar
jabza jabza ไม่อยู่ในระบบ
บัณฑิตฟ้า
 
วันที่สมัครสมาชิก: 02 สิงหาคม 2005
ข้อความ: 544
jabza is on a distinguished road
Default

กำ ทำไมมันยุ่งยาก จังเลยคับ


ให้พวก เทพๆ มาแสดงฝีมือกันหน่อยคับ
__________________
จะขอทำฝัน....ให้ใกล้เคียงความจริงที่สุด

เด็กน้อย ค่อยๆ เรียนรู้ สินะ
ตอบพร้อมอ้างอิงข้อความนี้
  #11  
Old 28 กรกฎาคม 2008, 19:11
Puriwatt's Avatar
Puriwatt Puriwatt ไม่อยู่ในระบบ
ลมปราณไร้สภาพ
 
วันที่สมัครสมาชิก: 14 กันยายน 2006
ข้อความ: 1,435
Puriwatt is on a distinguished road
Default

จากโจทย์ $m^3 + 6m^2 + 5m$ = $27n^3+9n^2+9n+1$

ผมคิดต่อโดยใช้สมการของคุณ MiRor (ที่คุณหยินหยางแก้ให้)
ได้รูปสมการใหม่คือ $m(m+1)(m+5)$ = $9n(3n^2+n+1)$ + $1$

จัดรูปใหม่ได้ $m(m+1)(m+2)$ + $3m(m+1)$ = $9n(3n^2+n+1)$ + $1$ ------ (1)

สมการ (1) $\div 3$ จะได้ $ \frac {m(m+1)(m+2)}{3}$ + $m(m+1)$ = $3n(3n^2+n+1)$ + $ \frac {1}{3}$ ------ (2)

L.H. เป็นจำนวนเต็มเสมอ เนื่องจาก m(m+1)(m+2) หารด้วย 3 ลงตัวแน่นอนครับ
R.H. ไม่เป็นจำนวนเต็ม เนื่องจากมีเศษ $ \frac {1}{3}$ เสมอ

ดังนั้นเราไม่มีทางหาจำนวนเต็ม m, n ใดๆ ที่ทำให้สมการข้างต้นเป็นจริงได้เลยครับ

29 กรกฎาคม 2008 00:21 : ข้อความนี้ถูกแก้ไขแล้ว 2 ครั้ง, ครั้งล่าสุดโดยคุณ Puriwatt
เหตุผล: ของเก่าดูยากขะรับกระผม
ตอบพร้อมอ้างอิงข้อความนี้
  #12  
Old 28 กรกฎาคม 2008, 23:03
หยินหยาง's Avatar
หยินหยาง หยินหยาง ไม่อยู่ในระบบ
กระบี่จักรวาล
 
วันที่สมัครสมาชิก: 06 มกราคม 2007
ข้อความ: 2,921
หยินหยาง is on a distinguished road
Default

อ้างอิง:
ข้อความเดิมเขียนโดยคุณ Puriwatt View Post
จากสมการของคุณ MiRor ที่คุณหยินหยางแก้ให้
จะได้รูปสมการใหม่คือ $9n(3n^2+n+1)$ = m(m+1)(m+5) - 1 = m(m+1)((m+2)+3) - 1
โจทย์เดิมเป็น $m^3+6m^2+5m = m(m+1)(m+5)$ ไม่ใช่ $m(m+1)(m+5) - 1$ ครับ
เพิ่มเติมอีกนิดครับ $9n(3n^2+n+1)$ อันนี้ก็ไม่ใช่ครับโจทย์จริงๆคือ $27n^3+9n^2+9n+1$ ครับ

ขอโทษครับเข้าใจความหมายที่จะอธิบายแล้วครับ
แต่ตอนที่ m(m+1)((m+2)+3) - 1 ต้องได้อย่างนี้ไม่ใช่หรือครับ m(m+1)(m+2)+3m(m+1) - 1

28 กรกฎาคม 2008 23:32 : ข้อความนี้ถูกแก้ไขแล้ว 2 ครั้ง, ครั้งล่าสุดโดยคุณ หยินหยาง
เหตุผล: เพิ่มเติมเพื่อความสมบูรณ์
ตอบพร้อมอ้างอิงข้อความนี้
  #13  
Old 29 กรกฎาคม 2008, 00:34
Puriwatt's Avatar
Puriwatt Puriwatt ไม่อยู่ในระบบ
ลมปราณไร้สภาพ
 
วันที่สมัครสมาชิก: 14 กันยายน 2006
ข้อความ: 1,435
Puriwatt is on a distinguished road
Default

อ้างอิง:
ข้อความเดิมเขียนโดยคุณ หยินหยาง View Post
แต่ตอนที่ m(m+1)((m+2)+3) - 1 ต้องได้อย่างนี้ไม่ใช่หรือครับ m(m+1)(m+2)+3m(m+1) - 1
ผมแก้ให้คุณหยินหยางแล้ว และจัดรูปแบบให้ดูง่ายขึ้นด้วยครับ

หมายเหตุ : วิธีแก้ปัญหาข้อ.1 ของคุณ jabza ทำให้ผมได้เข้าใจเรื่องสมการดีขึ้นมากเลย(เหมือนตื่นจากการหลับ)
และหวังว่าคุณ jabza คงจะเข้าใจข้อ.2 ที่ผมเสนอแนวคิดได้ไม่ยากนะครับ
ตอบพร้อมอ้างอิงข้อความนี้
  #14  
Old 29 กรกฎาคม 2008, 06:59
Yo WMU Yo WMU ไม่อยู่ในระบบ
ลมปราณคุ้มครองร่าง
 
วันที่สมัครสมาชิก: 15 กรกฎาคม 2008
ข้อความ: 265
Yo WMU is on a distinguished road
Default

ขอบคุณ คุณ Puriwatt มากครับ พอดีโจทย์ 2 ข้อที่ตั้งไว้เป็นโจทย์เพชรยอดมงกุฎ ม.ต้นปี 50 ครับ ข้อนี้เป็นช้อย (พอดีผมลืมให้ช้อยไป)
คือ 1) 0 คู่ 2) 1 คู่ 3) 3 คู่ 4) 9 คู่ และเฉลยตอบข้อ 1 (แต่ไม่เฉลยวิธีทำครับ)
ตอบพร้อมอ้างอิงข้อความนี้
ตั้งหัวข้อใหม่ Reply



กฎการส่งข้อความ
คุณ ไม่สามารถ ตั้งหัวข้อใหม่ได้
คุณ ไม่สามารถ ตอบหัวข้อได้
คุณ ไม่สามารถ แนบไฟล์และเอกสารได้
คุณ ไม่สามารถ แก้ไขข้อความของคุณเองได้

vB code is On
Smilies are On
[IMG] code is On
HTML code is Off
ทางลัดสู่ห้อง


เวลาที่แสดงทั้งหมด เป็นเวลาที่ประเทศไทย (GMT +7) ขณะนี้เป็นเวลา 05:32


Powered by vBulletin® Copyright ©2000 - 2024, Jelsoft Enterprises Ltd.
Modified by Jetsada Karnpracha